Chapter 40: tThe Child with a Musculoskeletal Disorder

Lakukan tugas rumah & ujian kamu dengan baik sekarang menggunakan Quizwiz!

The nurse is caring for an 8-year-old girl in traction. She has been in an acute care setting for two weeks and will require an additional 10 days in the hospital. She is showing signs of regression with thumb sucking and pleas for her tattered baby blanket. Which of the following would be the most helpful intervention? a) "Do you want a book to read?" b) "Let's ask your mom to bring your friends for a visit." c) "Would you like a coloring book?" d) "You are too big to suck your thumb."

"Let's ask your mom to bring your friends for a visit." Correct Explanation: After two weeks in traction, a child can become easily bored and regress in social and personal skills. A visit from friends arranged by the girl's mother or supervised by the child-life specialist would help her adapt to her immobilized state. Telling the girl she is too big to suck her thumb is unhelpful. Suggesting a book or coloring book would be unhelpful at this point, as she has likely grown tired of books and coloring after two weeks.

A nurse is caring for a 13-year-old boy with Duchenne muscular dystrophy. He says he feels isolated and that there is no one who understands the challenges of his disease. How should the nurse respond? a) "You need to remain as active as possible and have a positive attitude." b) "There are a lot of kids with the same type of muscular dystrophy you have at the MDA support group." c) "You have to go to a support group; it will be very helpful." d) "There are many things that you can do like crafts, computers or art."

"There are a lot of kids with the same type of muscular dystrophy you have at the MDA support group." The best response would be to remind the boy that there are many children with muscular dystrophy that could be found at the local support group. Teenagers do not like to be told that they "have" to do anything. Telling the boy that he needs to be active or simply suggesting activities does not address his concerns.

After teaching the parents of a 6-year-old child about caring for a sprained wrist, which statement by the parents indicates the need for additional teaching? a) "We'll apply a warm moist compress to the wrist for 20 minutes at a time." b) "We'll use an elastic wrap to keep the swelling down." c) "We'll be sure she keeps her wrist elevated above her heart."

"We'll apply a warm moist compress to the wrist for 20 minutes at a time." Correct Explanation: Care for a sprain includes rest, ice, compression, and elevation. Cold therapy, not heat, is used for 20 to 30 minutes at a time, then removed for 1 hour and repeated for the first 24 to 48 hours. Compression via an elastic bandage, elevating above heart level, and limiting activity are appropriate measures.

The nurse is taking the history of a 4-year-old boy. His mother mentions that he seems weaker and unable to keep up with his 6-year-old sister on the playground. Which question should the nurse ask to elicit the most helpful information? a) Would you please describe the weakness you are seeing in your son?" b) "Do you think he is simply fatigued?" c) "Has his pace of achieving milestones diminished?" d) "Has he achieved his developmental milestones on time?"

"Would you please describe the weakness you are seeing in your son?" The nurse needs to obtain a clear description of weakness. This open-ended question would most likely elicit specific examples of weakness and shed light on whether the boy is simply fatigued. The other questions would most likely elicit a yes or no answer rather than any specific details about his weakness or development.

A group of students are reviewing information about bone healing in children. The students demonstrate understanding of this information when they identify which of the following? a) A child's bones heal more quickly than those of an adult. b) The process of breaking down and forming new bone is decreased in children compared with adults. c) Callus production is slower but greater in amount in children than in adults. d) A fracture closer to the growth plate heals much slower than one in the metaphysis.

A child's bones heal more quickly than those of an adult. Correct Explanation: Bone healing occurs in the same fashion as in the adult, but it occurs more quickly in children because of the rich nutrient supply to the periosteum. The closer a fracture is to the growth plate, the more quickly the fracture heals. The capacity for remodeling (the process of breaking down and forming new bone) is increased in children compared with adults. Children's bones produce callus more rapidly and in larger quantities than do adults' bones.

A group of students are reviewing information about the skeletal development in children. The students demonstrate understanding of the information when they identify that ossification is complete by what age? a) Preschool age b) School age c) Adolescence d) Toddlerhood

Adolescence Correct Explanation: Ossification and conversion of cartilage to bone continue throughout childhood and are complete at adolescence.

You assist with the application of a full-body plaster cast to a child. The child immediately becomes diaphoretic and complains of being hot. Which nursing intervention would be indicated? a) Moisten the cast with cool water. b) Observe the child for infection. c) Advise the child that this is to be expected. d) Suggest removal of the cast to the orthopedist.

Advise the child that this is to be expected. Correct Explanation: Plaster becomes hot as it sets. This effect is reduced with newer plastic casts. This is a normal expectation about which to educate the child before the application of the cast. If discomfort continues, notify the provider. Infection would not present in this way with a cast application. Never moisten a case.

A nurse applies ice to a patient's leg to relieve the pain due to a soft tissue injury. Which of the following is a recommended guideline for use of cold therapy? a) Apply gel packs for no longer than 30 minutes and monitor closely for tissue damage. b) Apply ice for the first 24 to 48 hours after injury to reduce edema.

Apply ice for the first 24 to 48 hours after injury to reduce edema. Explanation: Application of ice is recommended for the first 24 to 48 hours after injury to reduce edema. Ice is usually applied as crushed ice in a bag, applied over a thin layer of cloth, or by immersing the injured part in cool water. Cold therapy is not recommended for persons with hypersensitivity to cold or with impaired circulation. Gel packs cool skin faster than an ice bag and should be applied for no more than 10 minutes.

A nurse is performing a physical examination of a child with a suspected fracture. Which assessment technique would the nurse be least likely to use? a) Palpation b) Observation c) Auscultation d) Inspection

Auscultation Explanation: The physical examination specific to fractures includes inspection, observation, and palpation. Auscultation is not used.

In understanding the development of the musculoskeletal system, the nurse recognizes that which of the following is implanted in a gel-like substance during fetal life? a) Ligaments b) Cartilage c) Joints d) Tendons

Cartilage Correct Explanation: During fetal life, tissue called cartilage, which is a type of connective tissue consisting of cells implanted in a gel-like substance, gradually calcifies and becomes bone.

The nurse is working with a group of caregivers of school-age children discussing fractures. The nurse explains that if the fragments of fractured bone are separated, the fracture is said to be which of the following? a) Incomplete b) Spiral c) Greenstick d) Complete

Complete Correct Explanation: If the fragments of fractured bone are separated, the fracture is said to be complete. If fragments remain partially joined, the fracture is termed incomplete. Green stick fractures are one kind of incomplete fracture, caused by incomplete ossification, common in children. Spiral fractures twist around the bone.

The nurse is caring for a 10-year-old girl in traction. The girl is experiencing muscle spasms associated with the traction. Which of the following would the nurse expect to administer if ordered? a) Alendronate b) Pamidronate c) Narcotic analgesics d) Diazepam

Diazepam Diazepam is an antianxiety drug that also has the effect of skeletal muscle relaxation; it is used for the treatment of muscle spasm associated with traction or casting. Narcotic analgesics are used for pain relief. Alendronate increases bone mineral density for children with osteogenesis imperfecta. Pamidronate increases bone mineral density for children with osteogenesis imperfecta.

The nurse is caring for a 10-year-old girl in traction. The girl is experiencing muscle spasms associated with the traction. Which of the following would the nurse expect to administer if ordered? a) Alendronate b) Pamidronate c) Narcotic analgesics d) Diazepam

Diazepam Explanation: Diazepam is an antianxiety drug that also has the effect of skeletal muscle relaxation; it is used for the treatment of muscle spasm associated with traction or casting. Narcotic analgesics are used for pain relief. Alendronate increases bone mineral density for children with osteogenesis imperfecta. Pamidronate increases bone mineral density for children with osteogenesis imperfecta.

The nurse is assessing a 10-year-old girl recently fitted with a cast on her wrist. Which assessment finding would alert the nurse to a possible infection? a) Diminished pulse b) Pallor of the fingers c) Drainage on the cast d) Delayed capillary refill

Drainage on the cast Correct Explanation: Drainage on the cast could indicate an infection. Pale fingers would suggest impaired circulation. Delayed capillary refill would suggest impaired circulation. Diminished pulse would suggest impaired circulation.

The nurse caring for a patient in a body cast knows that immobility can cause contractures, loss of muscle tone, or fixation of joints. Which of the following nursing interdisciplinary interventions are recommended to help prevent these adverse conditions? a) Check for a normal capillary refill of 3 to 5 seconds on a daily basis to ensure there in adequate arterial supply. b) Encourage active and passive range-of-motion activities to prevent ineffective tissue perfusion. c) Encourage child to stifle cough and take shallow breaths to prevent ineffective breathing patterns. d) Give the patient large, frequent meals with decreased fiber and increased protein and Vitamin C.

Encourage active and passive range-of-motion activities to prevent ineffective tissue perfusion. Correct Explanation: The nurse should turn the patient and encourage active and passive range-of-motion activities to prevent ineffective tissue perfusion. The patient should be instructed to cough and breathe deeply to prevent respiratory complications. Normal capillary refill is 1 to 3 seconds. The patient should be given small, frequent meals with increased fiber, protein, and vitamin C to prevent malnutrition.

Fracture of the femur typically occurs when a small child is lifted by one hand, as happens when a parent pulls on one arm to lift the child over a curb or up a step. a) True b) False

False Explanation: If a small child is lifted by one hand, as happens when a parent pulls on one arm to lift the child over a curb or up a step, the head of the radius may escape the ligament surrounding it and become dislocated (nursemaid's elbow). Fracture of the femur is rare and is typically caused by an automobile accident, a fall from a considerable height, or child maltreatment.

The nurse is conducting a physical examination of a 10-year-old boy with a suspected neuromuscular disorder. Which finding is a sign of Duchenne muscular dystrophy? a) Appearance of smaller than normal calf muscles b) Gowers sign c) Indications of hydrocephalus d) Lordosis

Gowers Sign A sign of Duchenne muscular dystrophy (DMD) is Gowers sign, or the inability of the child to rise from the floor in the standard fashion because of weakeness. Signs of hydrocephalus are not typically associated with DMD. Kyphosis and scoliosis occur more frequently than lordosis. A child with DMD has an enlarged appearance to their calf muscles due to pseudohypertrophy of the calves.

A type of traction sometimes used in the treatment of the child with scoliosis is called which of the following? a) Dunlop's traction b) Russell traction c) Halo traction d) Bryant's traction

Halo traction Correct Explanation: When a child has a severe spinal curvature or cervical instability, a form of traction known as halo traction may be used to reduce spinal curves and straighten the spine. Halo traction is achieved by using stainless steel pins inserted into the skull while counter-traction is applied by using pins inserted into the femur. Weights are increased gradually to promote correction.

An adolescent girl with scoliosis is refusing to wear the prescribed body brace. Which instruction is best to progress her to the treatment goals? a) It is important to wear the brace now to improve your spinal alignment, decreasing your symptoms . b) It is important to prevent herniation of a spinal disk, which is painful. c) It is important to prevent torticollis. d) It is important to correct spinal curvature before it gets too bad, causing you problems.

It is important to wear the brace now to improve your spinal alignment, decreasing your symptoms. It is important to have the adolescent understand the treatment and how the treatment will benefit them currently. Body bracing helps to hold the spine in alignment and prevent further curvature decreasing symptoms. The brace will not correct the problem. Herniation and torticollis are not associated with scoliosis.

The nurse is caring for a child with rickets. Which diagnostic test result would the nurse expect to find in the child's medical record? a) Low serum calcium levels b) X-ray confirmation of adequate bone shape c) Low alkaline phosphate levels d) High serum phosphate levels

Low serum calcium levels Correct Explanation: With rickets, serum calcium and phosphate levels are low and alkaline phosphate levels are elevated. Radiographs show changes in the shape and structure of the bone.

You meet a child with a slipped femoral epiphysis. In what type of child does this usually occur? a) Preadolescent girls b) Obese adolescent boys c) Tall, thin girls d) Active school-aged children

Obese adolescent boys Correct Explanation: A slipped epiphyseal femur injury most typically occurs in overweight preadolescent or adolescent boys. Stress increases the risk. A thin child would not have an increased risk, and the age range is past preadolescent and school age.

A nursing instructor is preparing a class presentation about tibia vara. Which of the following would the instructor include as a risk factor? a) Obesity b) Lack of sunlight exposure c) Late walking d) Hormonal alterations during puberty

Obesity Correct Explanation: Obesity is a risk factor for the development of tibia vara. Tibia vara occurs most frequently in children who are early walkers. Limited or lack of exposure to sunlight may lead to rickets. Hormonal alterations during puberty may play a role in the development of slipped capital femoral epiphysis.

The nurse is caring for a child admitted with Legg-Calvé-Perthes disease. Which of the following clinical manifestations would likely have been noted in the child with this diagnosis? a) Pain in the groin and a limp b) Difficulty standing and walking c) Poor posture and malformed vertebrae d) Inflammation of the joints

Pain in the groin and a limp Correct Explanation: Symptoms first noticed in Legg-Calvé-Perthes disease are pain in the hip or groin and a limp accompanied by muscle spasms and limitation of motion.

The nurse is doing neurovascular checks on a child who has had a cast applied to treat a fracture. The nurse observes for diminished or absent sensation and numbness or tingling. In doing this the nurse is monitoring for which of the following symptoms? a) Pallor b) Paralysis c) Pain d) Paresthesia

Paresthesia Correct Explanation: Paresthesia is diminished or absent sensation or numbness or tingling. Pallor is paleness of color and paralysis is the loss of function.

The nurse is conducting a physical examination of a 9-month-old infant with a suspected neuromuscular disorder. Which finding would warrant further evaluation? a) Presence of symmetrical spontaneous movement b) Absence of Moro reflex c) Presence of Moro reflex d) Absence of tonic neck reflex

Presence of Moro reflex The persistence of a primitive reflex in a 9-month-old would warrant further evaluation. Symmetrical spontaneous movement and absence of the Moro and tonic neck reflex are expected in a normally developing 9-month-old child.

The nurse is discussing types of treatment used when working with children who have orthopedic disorders. Which of the following forms of treatment covers the lower part of the body, usually from the waist down, and either one or both legs while leaving the feet open? a) External fixation device b) Stockinette c) Internal fixation device d) Spica cast

Spica cast Correct Explanation: The hip spica cast covers the lower part of the body, usually from the waist down, and either one or both legs while leaving the feet open. The cast maintains the legs in a frog-like position. Usually, there is a bar placed between the legs to help support the cast.

A neonatal nurse examines an infant and notes decreased hip motion that causes pain upon movement. This nurse suspects Legg-Calvé-Perthes disease, a common pediatric hip disorder that causes pain and decreased hip motion, possibly leading to a femoral head deformity. a) False b) True

True Correct Explanation: Legg-Calvé-Perthes disease is a common pediatric hip disorder that causes pain and decreased hip motion, possibly leading to a femoral head deformity. It has an incidence of 1 per 1,200 live births, with some hereditary factors influencing incidence.

The nurse is caring for a 3-year-old boy with a fracture of the humerus. His chart indicates "fracture is partially through the physis extending into the metaphysis." The nurse identifies this as which Salter-Harris classification? a) Type I b) Type V c) Type II d) Type IV

Type II Correct Explanation: According to the Salter-Harris classification, a type II fracture is partially through the physis extending into the metaphysis. A type I fracture is through the physis, widening it. A type IV fracture is through the metaphysis, physis, and epiphysis. A type V fracture is a crushing injury to the physis.

The nurse is caring for a 6-year-old boy with Russell traction applied to his left leg. Which intervention would be most appropriate to prevent complications? a) Assess the popliteal region carefully for skin breakdown. b) Provide pin care as needed. c) Adjust the weights as needed. d) Clean and massage his entire leg daily.

a)Assess the popliteal region carefully for skin breakdown. Explanation: The nurse would assess the popliteal region carefully for skin breakdown from the sling. The nurse would adjust the weights only per physician orders. Cleaning and massaging the skin is unrelated to care of the child with Russell traction. Russell traction is a form of skin traction, so there is no pin care.

Which diagnostic measure is most accurate in detecting neural tube defects? a) Presence of high maternal levels of albumin after 12th week of gestation b) Significant level of alpha-fetoprotein present in amniotic fluid c) Amniocentesis for lecithin-sphingomyelin (L/S) ratio d) Flat plate of the lower abdomen after the 23rd week of gestation

b)Significant level of alpha-fetoprotein present in amniotic fluid Screening for significant levels of alpha-fetoprotein is 90% effective in detecting neural tube defects. Prenatal screening includes a combination of maternal serum and amniotic fluid levels, amniocentesis, amniography, and ultrasonography and has been relatively successful in diagnosing the defect. Flat plate X-rays of the abdomen, L/S ratio, and maternal serum albumin levels aren't diagnostic for the defect.

An infant is placed in Bryant's traction. For Bryant's traction to be effective, the infant must be positioned on the a) back with hips flat on the bed. b) stomach with both legs extended. c) back with the injured hip flexed and the uninjured one extended. d) back with hips up off the bed.

back with hips up off the bed. Explanation: For there to be traction, the infant's hips must be off the bed. On the stomach or hips on the bed are not the correct positions for this patient.

A 9-year old child is scheduled for a computed tomography with contrast medium. Which of the following would be most important for the nurse to assess? a) Swelling b) Pain c) White blood cell count d) Allergies

d)Allergies Explanation: Assessing for allergies would be the priority because a contrast medium is being used. Pain is an important assessment but is unrelated to the test scheduled. Swelling is an important assessment finding, but this is unrelated to the test scheduled. Although a white blood cell count is important for determining an infection, it is unrelated to the test scheduled.

Through which mechanism is Duchenne's muscular dystrophy acquired? a) Virus b) Environmental toxins c) Autoimmune factors d) Heredity

d)Heredity Muscular dystrophy is hereditary and acquired through a recessive sex-linked trait. Therefore, it isn't caused by viral, autoimmune, or environmental factors

A girl with scoliosis is prescribed a body brace. The purpose of the brace is to a) improve spinal alignment. b) prevent herniation of a spinal disk. c) correct spinal curvature. d) prevent torticollis.

improve spinal alignment. Correct Explanation: Body bracing helps to hold the spine in alignment and prevent further curvature. The brace will not correct the problem. Herniation and torticollis are not associated with scoliosis.

A nurse is working with a 12-year-old girl with osteomyelitis who is recovering from surgery. Which of the following are nursing interventions that should be implemented in this case? *(Select all that apply.) a) Instituting infection-control precautions related to drainage tubes b) Administration of IV antibiotics at the hospital c) Instruction to the parents regarding how to care for an antibiotic IV line at home d) Casting of the affected limb e) Instruction to the parents regarding the importance of the child maintaining bed rest f) Instruction to the parents regarding proper traction of the limb

• Administration of IV antibiotics at the hospital • Instruction to the parents regarding how to care for an antibiotic IV line at home • Instruction to the parents regarding the importance of the child maintaining bed rest • Instituting infection-control precautions related to drainage tubes Explanation: Osteomyelitis is infection of the bone. Medical therapy includes limitation of weight bearing on the affected part, bed rest, immobilization, and a short administration of an IV antibiotic such as oxacillin (Bactocill), as indicated by the blood culture. Intravenous therapy is usually initiated in the hospital and then continued at home for as long as 2 weeks. When the child is discharged from the hospital, be certain to review with parents measures to care for the antibiotic intravenous line if this will be continued at home. Keep in mind young children are active, even if they are on bed rest so need age appropriate activities so they maintain rest, not activity. If a child had surgery and drainage tubes are in place, institute infection-control precautions, because the drain evacuates infected material. Neither casting nor traction is required for osteomyelitis.

The school nurse cares for children with overuse injuries and refers them for treatment. Which statements accurately describe conservative interventions to prevent or care for these types of injuries? Select all that apply. a) Avoid the causative activity for 6 to 8 weeks. b) Avoid using NSAIDs for pain control. c) Immobilize the muscles that are involved. d) Have the coach monitor the treatment program for sports injuries. e) Apply ice to the injured area to reduce inflammation. f) Link the chain of communication in the disciplinary approach.

• Avoid the causative activity for 6 to 8 weeks. • Apply ice to the injured area to reduce inflammation. • Link the chain of communication in the disciplinary approach. Explanation: Conservative treatment methods for the child with an overuse injury include avoiding the causative activity for 6 to 8 weeks and applying ice to the injured area to reduce the inflammation and irritation. NSAIDs (ibuprofen) are used for inflammation and pain control. The physical therapist institutes a stretching and strengthening program for the appropriate muscle groups. Parents and coaches may not understand that the level of activity that causes overuse symptoms varies from child to child. Notes or telephone conversations from the physician or nurse to the child's coach can clarify any misconceptions about what is expected during the recovery and recuperative periods.

The nurse is doing patient teaching with a child who has been placed in a brace to treat scoliosis. Which of the following statements made by the child indicates an understanding of the treatment? a) "At least when I take a shower I have a few minutes out of this brace." b) "I am so glad I can take this brace off for the school dance." c) "When I start feeling tired, I can just take my brace off for a few minutes." d) Wearing this brace only during the night won't be so embarrassing."

"At least when I take a shower I have a few minutes out of this brace." Correct Explanation: The brace worn to treat scoliosis is worn day and night and should be removed only very briefly, such as for showering. The child needs to be taught that the brace must be worn at all times, during the day as well as the night.

In discussing the treatment for children with scoliosis, a group of pediatric nurses makes the following statements. Which statement is most accurate related to the treatment of scoliosis? a) "The only successful treatment for scoliosis is surgery within two weeks of the diagnosis." b) "Children treated for scoliosis by using braces have to wear the brace almost all the time." c) "Children with severe scoliosis are treated using electrical stimulation." d) "The treatment for children with scoliosis usually lasts three to four months."

"Children treated for scoliosis by using braces have to wear the brace almost all the time." Correct Explanation: The Boston brace and the TLSO brace are made of plastic and are customized to fit the child for treatment of scoliosis. The brace should be worn constantly, except during bathing or swimming, to achieve the greatest benefit.

The nurse is reinforcing teaching with the caregivers of a child who has been placed in an external fixation device for the treatment of an orthopedic condition. Which of the following statements made by the caregivers indicate an understanding of the external fixation device? a) "If we see any drainage around the pins when we are cleaning them, we won't be concerned." b) "He is very sensitive about the way the device looks, I am glad that his clothes will fully cover it so his friends won't tease him." c) "We will have to get some of the elastic bandages to place around the pins and pin sites." d) "It will be hard, but we know our child will be in this device for a long time."

"It will be hard, but we know our child will be in this device for a long time." Correct Explanation: External fixation devices are sometimes left in place for as long as 1 year. The pin sites are left open to the air and should be inspected and cleansed every 8 hours. The child and caregiver should be able to recognize the signs of infection at the pin sites. The appearance of the pins puncturing the skin and the unusual appearance of the device can be upsetting to the child.

The caregiver of a 2-year-old who has a polyurethane resin cast on her arm calls the clinic to report that her child is crying and says that the cast has sand in it. The caregiver states that she has had casts of her own and knows how badly they can itch. She says she always used a hanger to scratch but is worried that it will be too sharp for the child. Which of the following statements would be appropriate for the nurse to make to this caregiver? a) "Since the child's cast is synthetic, she could soak it with cool water." b) "A plastic ruler is less likely than a hanger to cut the child's skin." c) "You could give the child an extra dose of acetaminophen and see if that helps." d) "Nothing should be put into the cast. You can blow cool air into it with a hair dryer."

"Nothing should be put into the cast. You can blow cool air into it with a hair dryer." Correct Explanation: Children and caregivers should be cautioned not to put anything inside the cast, no matter how much the casted area itches. Small toys and sticks or stick-like objects should be kept out of reach until the cast has been removed. Ice packs applied over the cast may help decrease the itching. Blowing cool air through a cast with a hair dryer set on a cool temperature or using a fan may help to relieve discomfort under a cast.

The nurse is teaching a group of peers regarding different types of fractures. Which of the following best describes an open fracture? a) A fracture in which there is a single break in the bone without penetration of the skin. b) A fracture in which the broken bone penetrates the skin. c) A fracture in which the fragments of the bone remain partially joined. d) A fracture in which the fragments of the bone are separated.

A fracture in which the broken bone penetrates the skin. Correct Explanation: The open fracture, also called a compound fracture, penetrates the skin. When the fragments of the bone are separated, the fracture is said to be complete.

A 3-year-old demonstrates lateral bowing of the tibia. Which of the following signs would indicate that the boy's condition is Blount disease rather than just the more typical developmental genu varum? a) A sharp, beaklike appearance to the medial aspect of the proximal tibia on x-ray b) The malleoli are touching c) The condition is bilateral d) The medial surfaces of the knees are more than 2 in apart

A sharp, beaklike appearance to the medial aspect of the proximal tibia on x-ray Correct Explanation: Blount disease is retardation of growth of the epiphyseal line on the medial side of the proximal tibia (inside of the knee) that results in bowed legs. Unlike the normal developmental aspect of genu varum, Blount disease is usually unilateral and is a serious disturbance in bone growth that requires treatment. In those with Blount disease, the medial aspect of the proximal tibia will show a sharp, beaklike appearance. The other answers all describe genu varum, not Blount disease.

The nurse is caring for an active 11-year-old presenting with tenderness in the shoulder. He is the pitcher for his baseball team and complains of shoulder pain with active internal rotation but is able to continue past the pain with full range of motion. Based on these reported symptoms, the nurse is aware that the disorder is most likely which of the following? a) Sever's disease b) Epiphysiolysis of the distal radius c) Osgood-Schlatter disease d) Epiphysiolysis of the proximal humerus

Epiphysiolysis of the proximal humerus Correct Explanation: Epiphysiolysis of the proximal humerus is an overuse disorder that occurs with rigorous upper extremity activity such as pitching and causes tenderness in the shoulder. Osgood-Schlatter disease causes knee pain and painful swelling or prominence of the anterior portion of the tibial tubercle. Sever disease causes pain over the posterior aspect of the calcaneus. Epiphysiolysis of the distal radius is an overuse disorder that causes wrist pain. It is common in gymnasts.

The nurse is caring for a 10-year-old child in traction. After performing a skin assessment, she notices that the skin over the calcaneus appears slightly red and irritated. Which of the following should be the first intervention? a) Reposition the child's foot on a pressure-reducing device. b) Gently massage his foot and heel each shift. c) Apply lotion to his foot and avoid friction to the area. d) Make sure the skin and linens are clean and dry.

Reposition the child's foot on a pressure-reducing device. Correct Explanation: The nurse's first action is to remove continuous pressure from this area. The other actions can help decrease potential for skin breakdown, but the pressure must be relieved first.

A nurse is assessing a newborn and observes webbing of the fingers and toes. The nurse documents this finding as which of the following? a) Polydactyly b) Pectus carinatum c) Metatarsus adductus d) Syndactyly

Syndactyly Correct Explanation: Syndactyly refers to webbing of the fingers and toes. Polydactyly refers to the presence of extra digits on the hand or foot. Metatarsus adductus is a medial deviation of the forefoot. Pectus carinatum is a protuberance of the chest wall.

A nurse is assisting the parents of a child who requires a Pavlik harness. The parents are apprehensive about how to care for their baby and concerned about holding and playing with him. How can the nurse best assist the parents? a) "Let's put you in touch with other families who have experienced this." b) "The baby only needs the harness for 12 weeks." c) "Do not attempt to adjust the harness yourself." d) "The harness does not hurt the baby."

a)"Let's put you in touch with other families who have experienced this." Explanation: There are many helpful pointers and suggestions that are available from other parents and orthopedic organizations. Referring the parents to other families who have experienced a Pavlik harness will provide assurance and likely increase compliance with the regimen. The other responses are factual but do not address the parent's concerns.

A nurse is working with a child who has Osgood-Schlatter disease. Which of the following clients would be the most likely to develop this condition? a) A 13-year-old boy who is on his school's cross-country team b) A 15-year-old girl who dances ballet c) A 9-year-old boy who is sedentary d) An 11-year-old girl who is obese

a)A 13-year-old boy who is on his school's cross-country team Explanation: Osgood-Schlatter disease is the thickening and enlargement of the tibial tuberosity resulting from microtrauma, probably caused from overuse. It occurs more often in boys than girls and at preadolescence or early adolescence, probably because of rapid growth at these times.

An infant appears floppy, sucks poorly, and has weak respiratory effort. She is diagnosed with congenital myasthenia gravis. In explaining this condition to the infant's mother, which of the following should she mention as the cause of the condition? a) An inherited disorder involving faulty neurotransmitter transmission or reception b) Infection of the bone by Streptococcus pyogenes c) Degeneration of skeletal muscle fibers, possibly due to viral infection d) An autoimmune disorder involving the development of circulating antibodies against body cells

a)An inherited disorder involving faulty neurotransmitter transmission or reception Explanation: With myasthenia gravis, there is interference in ACh processing, which leads to symptoms of progressive muscle weakness or inability to contract. Congenital myasthenia appears to be an inherited disorder that results in faulty ACh transmission or reception. Dermatomyositis occurs from degeneration of skeletal muscle fibers. The cause of the disorder is unknown, although either a viral infection or an autoimmune basis is suspected. The cause of juvenile arthritis is unknown, although it is thought to be an autoimmune process in which a child develops circulating antibodies (immunoglobulins) against body cells. Osteomyelitis is infection of the bone by Streptococcus pyogenes.

A nurse is applying a cast to a 12-year-old boy with a simple fracture of the radius in the arm. Which of the following is most important for the nurse to do when she has finished applying the cast? a) Assess the fingers for warmth, pain, and function b) Apply a tube of stockinette over the cast c) Cut a window in the cast over the wrist d) X-ray the cast to make sure the bones are aligned properly

a)Assess the fingers for warmth, pain, and function Explanation: Assess fingers or toes carefully for warmth, pain, and function after application of a cast to be certain a compartment syndrome is not developing. Before a cast is applied, not after, a tube of stockinette is stretched over the area, and soft cotton padding is placed over bony prominences. A "window" may be placed in a cast for an open fracture or if an infection is suspected—not to prevent an infection—so that the area can be observed; however, a window is not indicated in this case. The x-ray should be performed before casting, to diagnose the fracture, not afterward.

The nurse is caring for a child with rickets. Which diagnostic test result would the nurse expect to find in the child's medical record? a) Low serum calcium levels b) Low alkaline phosphate levels c) High serum phosphate levels d) X-ray confirmation of adequate bone shape

a)Low serum calcium levels Explanation: With rickets, serum calcium and phosphate levels are low and alkaline phosphate levels are elevated. Radiographs show changes in the shape and structure of the bone.

The nurse caring for a client with suspected muscular dystrophy would prepare her client for which diagnostic test? a) Muscle biopsy b) EEG c) Assessment of ambulation d) X-ray

a)Muscle biopsy A muscle biopsy shows the degeneration of muscle fibers and infiltration of fatty tissue. It's used for diagnostic confirmation of muscular dystrophy. X-ray is best for identifying an osseous deformity. Ambulation assessment alone wouldn't confirm diagnosis of this client's disorder. EEG wouldn't be appropriate in this case.

You meet a child with a slipped femoral epiphysis. In what type of child does this usually occur? a) Obese adolescent boys b) Active school-aged children c) Tall, thin girls d) Preadolescent girls

a)Obese adolescent boys Explanation: A slipped epiphyseal femur injury most typically occurs in overweight preadolescent or adolescent boys. Stress increases the risk. A thin child would not have an increased risk, and the age range is past preadolescent and school age.

A neonatal nurse examines an infant and notes decreased hip motion that causes pain upon movement. This nurse suspects Legg-Calvé-Perthes disease, a common pediatric hip disorder that causes pain and decreased hip motion, possibly leading to a femoral head deformity. a) True b) False

a)True Explanation: Legg-Calvé-Perthes disease is a common pediatric hip disorder that causes pain and decreased hip motion, possibly leading to a femoral head deformity. It has an incidence of 1 per 1,200 live births, with some hereditary factors influencing incidence.

The nurse is caring for a 3-year-old boy with a fracture of the humerus. His chart indicates "fracture is partially through the physis extending into the metaphysis." The nurse identifies this as which Salter-Harris classification? a) Type II b) Type V c) Type I d) Type IV

a)Type II Explanation: According to the Salter-Harris classification, a type II fracture is partially through the physis extending into the metaphysis. A type I fracture is through the physis, widening it. A type IV fracture is through the metaphysis, physis, and epiphysis. A type V fracture is a crushing injury to the physis.

An infant is placed in Bryant's traction. For Bryant's traction to be effective, the infant must be positioned on the a) back with hips up off the bed. b) stomach with both legs extended. c) back with the injured hip flexed and the uninjured one extended. d) back with hips flat on the bed.

a)back with hips up off the bed. Correct Explanation: For there to be traction, the infant's hips must be off the bed. On the stomach or hips on the bed are not the correct positions for this patient.

The nurse is talking with the caregiver of a 13-year-old diagnosed with scoliosis. The child has come to the clinic to be fitted with a brace to begin her treatment. The child appears upset and angry and states, "I hate this brace; I hate it already." In an effort to support this child, which statement would be the most appropriate for the nurse to make to this child's caregiver? a) "Remind your child that her spine needs to be corrected in order to keep her whole musculoskeletal system healthy for a long, long time." b) "Children her age often withdraw during stressful times; let her have some time alone to think about the situation and to get used to the brace." c) "Take your daughter to an oncology floor for a few minutes so she can see children who are much sicker than she is." d) "If you can afford it, let your daughter choose an article or two of clothing that she can wear with the brace that will help her feel that she looks good."

b)"If you can afford it, let your daughter choose an article or two of clothing that she can wear with the brace that will help her feel that she looks good." Help the child select clothing that blends with current styles but is loose enough to hide the brace. Self-image and the need to be like others are very important at this age. Wearing a brace creates a distinct change in body image, especially in the older child or adolescent, at a time when body consciousness is at an all-time high. The need to wear the brace and deal with the limitations it involves may cause anger; the change in body image can cause a grief reaction. Handling these feelings successfully requires understanding support from the nurse, family, and peers. It is important for the child to have an opportunity to talk about his or her feelings.

The nurse caring for a child who has been put into a leg cast must be on the alert for signs of nerve and muscle damage. Which of the following symptoms might be an early warning signal that the child has developed compartment syndrome? The child a) Has a weak femoral pulse b) Feels increasing severe pain c) Cannot plantarflex his foot d) Has blue-looking nail beds on the toes

b)Feels increasing severe pain Explanation: Any complaint of pain in a child with a new cast or immobilized extremity needs to be explored and monitored closely for the possibility of compartment syndrome.

Which nursing diagnosis will the nurse prepare for the infant who is placed prone to protect the myelomeningocele repair site? a) Disorganized infant behavior b) Risk for impaired skin integrity c) Peripheral neurovascular dysfunction d) Risk for activity intolerance

b)Risk for impaired skin integrity The skin of the infant's knees and elbows is exposed to both pressure and friction. Leakage of urine and stool makes skin cleanliness a challenge. Should voluntary movement of the legs be affected, they become more vulnerable to skin integrity problems. The neuromuscular dysfunction the infant experiences is neither peripheral nor vascular. Disorganized infant behavior does not reflect the reality of the situation, and risk for activity intolerance is not appropriate because little activity occurs.

The nurse performing a focused health history on a newborn asks the parents if there are any hereditary disorders affecting musculoskeletal function in the family history. These disorders include (select all answers that apply): a) Hip dysplasia b) Scoliosis c) Clubfoot d) Brachial plexus injury e) Limb deformities

b)Scoliosis c)Clubfoot a)Hip dysplasia Explanation: The nurse should explore the family history for any hereditary disorders. The presence of scoliosis, clubfoot, hip or skeletal dysplasia, or neuromuscular disorders in family members may help in diagnosing genetically linked orthopedic disorders.

Structural scoliosis is five times more common in girls than in boys. a) False b) True

b)True

You are caring for a child with a broken wrist that has just been placed in a cast. You would elevate the arm to a) promote healing. b) prevent edema. c) ensure proper bone alignment. d) discourage infection.

b)prevent edema. Explanation: Edema tends to be dependent. Elevating the arm, therefore, would reduce swelling from the injury. Elevation of the arm would not promote healing nor discourage infection. The cast will maintain proper bone alignment.

The nurse is caring for a newborn with facial nerve palsy from birth trauma. The mother is very upset and concerned about the child's prognosis. Which response by the nurse would be most appropriate? a) "Have you seen any signs of improvement?" b) "This is the most common facial nerve palsy." c) "In most cases treatment is not necessary, only observation." d) "Was this from pressure resulting from forceps?"

c)"In most cases treatment is not necessary, only observation." The nurse should reassure the mother by reminding her that in most cases treatment is not necessary, only observation. Asking about signs of improvement might alarm the mother because in some cases it can take many months for the palsy to resolve. Asking whether this was a result of pressure from forceps does not address the mother's concerns about the child's prognosis. The mother may not understand or know why the condition occurred. Telling the mother that this is the most common facial nerve palsy does not address the mother's concerns about the child's prognosis.

A 14-year-old boy was diagnosed with a closed fracture of the ulna at approximately 9 a.m. The fracture was reduced in the emergency room and his arm placed in a cast. At 6 p.m. his mother has brought him back to the emergency room due to unrelenting pain that has not been relieved by his prescribed narcotics. What should the nurse do first? a) Assess the neurovascular status of the arm. b) Elevate the arm above the level of the heart. c) Alert the doctor immediately and apply ice. d) Give additional pain medication as ordered.

c)Alert the doctor immediately and apply ice. Explanation: The nurse should notify the doctor immediately because the boy's symptoms are the classic sign of compartment syndrome. Immediate treatment is required to prevent excessive swelling and to detect neurovascular compromise as quickly as possible.

A nurse applies ice to a patient's leg to relieve the pain due to a soft tissue injury. Which of the following is a recommended guideline for use of cold therapy? a) Use cold therapy to increase blood flow in patients with impaired circulation. b) Apply gel packs for no longer than 30 minutes and monitor closely for tissue damage. c) Apply ice for the first 24 to 48 hours after injury to reduce edema. d) Apply the ice to the injury, never immerse the injured part in cold water.

c)Apply ice for the first 24 to 48 hours after injury to reduce edema. Explanation: Application of ice is recommended for the first 24 to 48 hours after injury to reduce edema. Ice is usually applied as crushed ice in a bag, applied over a thin layer of cloth, or by immersing the injured part in cool water. Cold therapy is not recommended for persons with hypersensitivity to cold or with impaired circulation. Gel packs cool skin faster than an ice bag and should be applied for no more than 10 minutes.

When performing physical assessments of children with musculoskeletal disorders, the nurse distinguishes normal variations in children's muscles versus adult muscles. These variations include: a) Rapid bone and muscle growth in adolescents increase their agility, thereby decreasing the incidence of injuries. b) The young child has rigid soft tissue, so dislocations and sprains are common occurrences. c) During adolescence, muscle growth is influenced by increased production of androgenic hormones. d) The infant's muscles account for 45% of total body weight as opposed to 25% of adult body weight.

c)During adolescence, muscle growth is influenced by increased production of androgenic hormones. Explanation: During adolescence, muscle growth is influenced by hormonal changes, primarily the increased production of androgenic hormones. The infant's muscles account for only 25% of total body weight, whereas they account for 40% to 45% in an adult. The young child has resilient soft tissue, so dislocations and sprains are unusual occurrences. Rapid bone and muscle growth may contribute to the appearance of "clumsy" and awkward motions of the adolescent who is trying to adjust to new body dimensions.

The nurse caring for a patient in a body cast knows that immobility can cause contractures, loss of muscle tone, or fixation of joints. Which of the following nursing interdisciplinary interventions are recommended to help prevent these adverse conditions? a) Encourage child to stifle cough and take shallow breaths to prevent ineffective breathing patterns. b) Give the patient large, frequent meals with decreased fiber and increased protein and Vitamin C. c) Encourage active and passive range-of-motion activities to prevent ineffective tissue perfusion. d) Check for a normal capillary refill of 3 to 5 seconds on a daily basis to ensure there in adequate arterial supply.

c)Encourage active and passive range-of-motion activities to prevent ineffective tissue perfusion. Explanation: The nurse should turn the patient and encourage active and passive range-of-motion activities to prevent ineffective tissue perfusion. The patient should be instructed to cough and breathe deeply to prevent respiratory complications. Normal capillary refill is 1 to 3 seconds. The patient should be given small, frequent meals with increased fiber, protein, and vitamin C to prevent malnutrition.

The nurse is caring for an extremely active 13-year-old girl who has recently been prescribed a back brace to treat scoliosis. Which of the following interventions will be most critical to the success of her treatment? a) Asking for, and listening to, the child's feelings about the treatment b) Showing the girl how the brace works and when to wear it c) Ensuring treatment compliance due to mobility limitations d) Teaching the parents about the disease and its treatment

c)Ensuring treatment compliance due to mobility limitations Explanation: Ensuring compliance with the treatment regime will ultimately be most critical. Educating the parents about scoliosis, showing the girl how to use the brace, and listening to the child's concerns are also important. But in the end, the most important factor is that the child wears the brace according to the treatment plan.

The nurse is caring for an active 11-year-old presenting with tenderness in the shoulder. He is the pitcher for his baseball team and complains of shoulder pain with active internal rotation but is able to continue past the pain with full range of motion. Based on these reported symptoms, the nurse is aware that the disorder is most likely which of the following? a) Sever's disease b) Epiphysiolysis of the distal radius c) Epiphysiolysis of the proximal humerus d) Osgood-Schlatter disease

c)Epiphysiolysis of the proximal humerus Explanation: Epiphysiolysis of the proximal humerus is an overuse disorder that occurs with rigorous upper extremity activity such as pitching and causes tenderness in the shoulder. Osgood-Schlatter disease causes knee pain and painful swelling or prominence of the anterior portion of the tibial tubercle. Sever disease causes pain over the posterior aspect of the calcaneus. Epiphysiolysis of the distal radius is an overuse disorder that causes wrist pain. It is common in gymnasts.

The mother of a 3-year-old with a myelomeningocele is thinking about having another baby. The nurse should inform the woman that she should increase her intake of which acid? a) Folic acid to 0.4 mg/day b) Ascorbic acid to 4 mg/day c) Folic acid above 0.4 mg/day d) Ascorbic acid to 0.4 mg/day

c)Folic acid above 0.4 mg/day The American Academy of Pediatrics recommends that a woman who has had a child with a neural tube defect increase her intake of folic acid to above 0.4 mg per day 1 month before becoming pregnant and continue this regimen through the first trimester. A woman who has no family history of neural tube defects should take 0.4 mg/day. All women of childbearing age should be encouraged to take a folic acid supplement because the majority of pregnancies in the United States are unplanned. Ascorbic acid hasn't been shown to have any effect on preventing neural tube defects.

A type of traction sometimes used in the treatment of the child with scoliosis is called which of the following? a) Bryant's traction b) Dunlop's traction c) Halo traction d) Russell traction

c)Halo traction Explanation: When a child has a severe spinal curvature or cervical instability, a form of traction known as halo traction may be used to reduce spinal curves and straighten the spine. Halo traction is achieved by using stainless steel pins inserted into the skull while counter-traction is applied by using pins inserted into the femur. Weights are increased gradually to promote correction.

A nurse is caring for a 10-year-old patient who is in skeletal traction following injuries sustained in a car accident. Which of the following accurately describes a recommended nursing measure for this type of traction? a) Use warm saline as a cleansing solution before and after pin care. b) Assess the traction weights daily and remove or add weight as necessary. c) Perform pin-site care on a daily or weekly basis after the first 48 to 72 hours. d) Make sure the weights are touching the floor or top of bed at all times.

c)Perform pin-site care on a daily or weekly basis after the first 48 to 72 hours. Explanation: At sites with mechanically stable bone-pin interfaces, pin-site care should be done on a daily or weekly basis (after the first 48 to 72 hours). The nurse should never remove or add traction weights without specific physician orders, or allow weights to touch the floor or drag on the bed parts; weights should hang free. A chlorhexidine 2 mg/mL solution may be the most effective cleansing solution for pin care.

A nurse is assessing a newborn and observes webbing of the fingers and toes. The nurse documents this finding as which of the following? a) Pectus carinatum b) Metatarsus adductus c) Syndactyly d) Polydactyly

c)Syndactyly Explanation: Syndactyly refers to webbing of the fingers and toes. Polydactyly refers to the presence of extra digits on the hand or foot. Metatarsus adductus is a medial deviation of the forefoot. Pectus carinatum is a protuberance of the chest wall.

The nurse is caring for a child in a type of traction in which weights are being used. Which of the following is true regarding the weights? a) The weights should be removed once a shift and then replaced. b) Additional weights will be added as the fracture is healing, usually once a day. c) The weights must be hanging freely, not touching the bed or floor. d) The weights can be removed and the child encouraged to move around in bed several times a day.

c)The weights must be hanging freely, not touching the bed or floor. Explanation: When a child is in traction the weights must be hanging freely, not touching the bed or floor.

When helping parents plan care for a child with Legg-Calvé-Perthes disease, you would teach them that the usual therapy for children with this disorder is a) surgery with supporting rods. b) exercise to increase muscle strength of the knee joint. c) a nonweight-bearing period. d) passive range-of-motion exercises TID.

c)a nonweight-bearing period. Explanation: Resting the affected femoral epiphysis aids healing.

The nurse is caring for a 10-year-old boy who plays on two soccer teams. He practices four days a week and his team travels to tournaments once a month. He has been diagnosed with a stress fracture in one of his vertebrae. Which of the following instructions is most important to emphasize to the boy and his parents? a) "Ice will help reduce the inflammation." b) "NSAIDs can help with pain control and inflammation." c) "You will need to see a physical therapist for stretching and strengthening exercises." d) "You and your coaches need to understand that you cannot play soccer for at least six weeks."

d)"You and your coaches need to understand that you cannot play soccer for at least six weeks." Explanation: A child with an overuse injury needs to avoid the causative activity for six to eight weeks. The other suggestions are also important, but the nurse must emphasize to the boy and his parents that they must tell the coaches "no soccer for six weeks." In some situations, it is helpful to supply a written directive from the nurse or physician to help the parent avoid undue pressure from coaches.

Maria is a 9-month-old whose babysitter brings her to the ER. An x-ray shows a spiral fracture of the femur. The babysitter tells the nurse that she found the infant in this condition when she showed up to watch her an hour ago. How should the nurse respond to this situation? a) Evaluate the child for an underlying musculoskeletal disorder. b) Call social services to find the parents and evaluate them for child abuse. c) Ask the babysitter to advocate for the child and report the incident to the authorities. d) Arrange for the parents to come in for an evaluation for possible physical abuse.

d)Arrange for the parents to come in for an evaluation for possible physical abuse. Correct Explanation: A child younger than age 1 who presents with a fracture should be evaluated for possible physical abuse. The parents should be contacted first, and the family should undergo an evaluation for possible physical abuse since femoral fractures in nonambulating infants, particularly spiral fractures, are believed to be highly specific for inflicted injury. If physical abuse is not found, the infant should be evaluated for an underlying musculoskeletal disorder.

A nurse caring for a child wearing a brace to correct scoliosis provides patient and family teaching for home care of the brace. Which of the following is an accurate intervention for this situation? Select all that apply. a) Schedule brace wear for waking hours for best therapeutic results. b) Tell the patient to loosen the brace during meals if necessary. c) Gradually decrease wearing time so the skin can develop tolerance. d) Avoid sitting in one position for long periods of time. e) Recommend a shower instead of a bath to stimulate the skin. f) Wear a 100%-cotton T-shirt under the brace to absorb moisture.

d)Avoid sitting in one position for long periods of time. b)Tell the patient to loosen the brace during meals if necessary. f)Wear a 100%-cotton T-shirt under the brace to absorb moisture. Explanation: The nurse should teach the patient to avoid sitting in one position for long periods of time and to loosen the brace during meals. The nurse should also encourage wearing the brace during sleep to use up the most hours and tell the family to increase wearing time gradually so that the skin can develop tolerance for the pressure of the brace. A 100%-cotton shirt should be worn under the brace, and a shower or bath should be taken daily.

The nurse is caring for a child after an accident in which the child fractured his arm. A cast has been applied to the child's right arm. Which of the following actions should the nurse implement? Select all that apply. a) Monitor the color of the nail beds in the right hand. b) Document any signs of pain. c) Wear a protective gown when moving the child's arm. d) Check radial pulse in the both arms. e) Wear sterile gloves when removing or touching the cast.

d)Check radial pulse in the both arms. b)Document any signs of pain. a)Monitor the color of the nail beds in the right hand. Correct Explanation: Monitoring for signs of pain, decreased circulation, or change or variation in pulses in the extremity is important for the child in a cast. Pain can indicate serious complications, such as compartment syndrome. Wearing a gown or sterile gloves is unnecessary. Checking posterior pulses would be appropriate when a lower extremity is casted.

The nurse caring for a client diagnosed with muscular dystrophy would expect which laboratory values to be most abnormal? a) Serum potassium b) Sodium c) Bilirubin d) Creatinine

d)Creatinine Creatinine is a by-product of muscle metabolism as the muscle hypertrophies. Bilirubin is a by-product of liver function. Potassium and sodium levels can change due to various factors and aren't indicators of muscular dystrophy.

The nurse is assessing a 10-year-old girl recently fitted with a cast on her wrist. Which assessment finding would alert the nurse to a possible infection? a) Diminished pulse b) Delayed capillary refill c) Pallor of the fingers d) Drainage on the cast

d)Drainage on the cast Explanation: Drainage on the cast could indicate an infection. Pale fingers would suggest impaired circulation. Delayed capillary refill would suggest impaired circulation. Diminished pulse would suggest impaired circulation.

The type of fracture often seen in young children is a fracture in which there is not complete ossification of the bone and the bone bends and just partially breaks. This type of fracture is which of the following? a) Epiphyseal b) Spiral c) Complete d) Greenstick

d)Greenstick Explanation: Greenstick fractures are one kind of incomplete fracture, caused by incomplete ossification, common in children. The bone bends and often just partially breaks.

The nurse is caring for a child who has just had a plaster cast applied to the arm. The nurse is correct in doing which of the following with this child? a) Keeping a clove-hitch restraint gently tied on the hand to stabilize the arm. b) Encouraging the child to move the arm slowly up and down to help the cast dry. c) Using only a draw sheet to move the casted arm. d) Handling the cast with open palms when moving the arm.

d)Handling the cast with open palms when moving the arm. Explanation: A wet plaster cast should be handled only with open palms because fingertips can cause indentations and result in pressure points. There is no reason the arm should be restrained or the arm moved to aid in the drying process.

In understanding the function of the musculoskeletal system, the nurse recognizes that which of the following allows for movement of the body parts? a) Cartilage b) Ligaments c) Tendons d) Joints

d)Joints Explanation: Bones are attached to each other by connecting links called joints, which allow for movement of the body parts. Skeletal muscles attach to the bones, with a moveable joint between them. Tendons and ligaments hold the muscles and bones together. Cartilage is a type of connective tissue consisting of cells implanted in a gel-like substance, which gradually calcifies and becomes bone.

The child diagnosed with muscular dystrophy often exhibits a forward curvature of the lumbar spine. This description is accurate regarding which of the following? a) Kyphosis b) Scoliosis c) Synovitis d) Lordosis

d)Lordosis Explanation: Lordosis, a forward curvature of the lumbar spine or swayback, is seen by school age in the child with muscular dystrophy.

The nurse is caring for a child admitted with Legg-Calvé-Perthes disease. Which of the following clinical manifestations would likely have been noted in the child with this diagnosis? a) Poor posture and malformed vertebrae b) Difficulty standing and walking c) Inflammation of the joints d) Pain in the groin and a limp

d)Pain in the groin and a limp Explanation: Symptoms first noticed in Legg-Calvé-Perthes disease are pain in the hip or groin and a limp accompanied by muscle spasms and limitation of motion.

The nurse is doing neurovascular checks on a child who has had a cast applied to treat a fracture. The nurse observes for diminished or absent sensation and numbness or tingling. In doing this the nurse is monitoring for which of the following symptoms? a) Pallor b) Pain c) Paralysis d) Paresthesia

d)Paresthesia Explanation: Paresthesia is diminished or absent sensation or numbness or tingling. Pallor is paleness of color and paralysis is the loss of function.

A nurse is preparing a plan of care for an infant who has undergone surgery to repair a myelomeningocele. The nurse would include placing the infant in which position postoperatively? Select all that apply. a) Left side lying b) Supine c) Semi-Fowler d) Prone e) Right side lying

d)Prone e)Right side lying a)Left side lying Postoperatively, the nurse would position the infant in the prone or side-lying position to allow the incision to heal.

The nurse is caring for a 10-year-old child in traction. After performing a skin assessment, she notices that the skin over the calcaneus appears slightly red and irritated. Which of the following should be the first intervention? a) Make sure the skin and linens are clean and dry. b) Gently massage his foot and heel each shift. c) Apply lotion to his foot and avoid friction to the area. d) Reposition the child's foot on a pressure-reducing device.

d)Reposition the child's foot on a pressure-reducing device. Explanation: The nurse's first action is to remove continuous pressure from this area. The other actions can help decrease potential for skin breakdown, but the pressure must be relieved first.

The nurse is reinforcing discharge teaching with the caregivers of a child who is going home after a cast has been applied. The nurse explains to the caregivers that which of the following should be reported if they occur or are seen related to this child? (Select all that apply) a) A foul odor under the cast b) Drainage from under the cast c) Any pink color in the fingers or toes of casted extremity d) Looseness of the cast on the extremity e) Any area on the cast that is warm to the touch f) Any itching under or around the edges of the cast

e)Any area on the cast that is warm to the touch a)A foul odor under the cast b)Drainage from under the cast d)Looseness of the cast on the extremity Explanation: In addition to the five Ps, any foul odor or drainage on or under the cast, "hot spots" on the cast (areas warm to the touch), looseness or tightness, or any el evation of temperature must be noted, documented, and re ported. Family caregivers should be instructed to watch carefully for these same danger signals. Itching is common and does not need to be reported. Pink coloration of fingers and toes would be normal and not a concern.

While an adolescent wears a body brace for scoliosis, you would teach her a) that secondary sex changes will stop until the brace is removed. b) to continue with age-appropriate activities. c) to wear the brace a maximum of 20 hours each day. d) to stand absolutely still whenever she is out of the brace.

to continue with age-appropriate activities. Correct Explanation: Wearing a body brace should not interfere with normal activities, which are necessary to maintain adolescent self-esteem. Sex changes continue with or without bracing; the provider will determine the length of time for wearing the brace each day.

The nurse is caring for a 14-year-old boy in Buck traction for a slipped capital femoral epiphysis (SCFE). Which of the following would the nurse include when completing a neurovascular assessment of the affected leg? Select all that apply. a) Sensation b) Vital signs c) Color d) Capillary refill e) Pulse

• Color • Sensation • Pulse • Capillary refill Explanation: A neurovascular assessment includes assessing for color, movement, sensation, edema, and quality of pulses. Vital signs are not a component of a neurovascular assessment.

The nurse is reinforcing discharge teaching with the caregivers of a child who is going home after a cast has been applied. The nurse explains to the caregivers that which of the following should be reported if they occur or are seen related to this child? (Select all that apply) a) Drainage from under the cast b) Any pink color in the fingers or toes of casted extremity c) A foul odor under the cast d) Any area on the cast that is warm to the touch e) Any itching under or around the edges of the cast f) Looseness of the cast on the extremity

• Drainage from under the cast • A foul odor under the cast • Any area on the cast that is warm to the touch • Looseness of the cast on the extremity Correct Explanation: In addition to the five Ps, any foul odor or drainage on or under the cast, "hot spots" on the cast (areas warm to the touch), looseness or tightness, or any el evation of temperature must be noted, documented, and re ported. Family caregivers should be instructed to watch carefully for these same danger signals. Itching is common and does not need to be reported. Pink coloration of fingers and toes would be normal and not a concern.

A nurse is assisting the parents of a child who requires a Pavlik harness. The parents are apprehensive about how to care for their baby and concerned about holding and playing with him. How can the nurse best assist the parents? a) "The baby only needs the harness for 12 weeks." b) "Do not attempt to adjust the harness yourself." c) "Let's put you in touch with other families who have experienced this." d) "The harness does not hurt the baby."

"Let's put you in touch with other families who have experienced this." Correct Explanation: There are many helpful pointers and suggestions that are available from other parents and orthopedic organizations. Referring the parents to other families who have experienced a Pavlik harness will provide assurance and likely increase compliance with the regimen. The other responses are factual but do not address the parent's concerns.

The nurse is caring for an infant girl in an outpatient setting. The infant has just been diagnosed with developmental dysplasia of the hip (DDH). The mother is very upset about the diagnosis and blames herself for her daughter's condition. Which response best addresses the mother's concerns? a) "This is not your fault and we will help you with her care and treatment." b) "Your daughter will likely wear a Pavlik harness." c) "There are simple noninvasive treatment options." d) "Don't worry; this is a relatively common diagnosis."

"This is not your fault and we will help you with her care and treatment." Because the mother is crying and experiencing the initial shock of the diagnosis, the nurse's primary concern is to support the mother and assure her that she is not to blame for the DDH. While education is important, the nurse should let the mother adjust to the diagnosis and assure her that the baby and her family will be supported now and throughout the treatment period

The nurse is caring for a 10-year-old boy who plays on two soccer teams. He practices four days a week and his team travels to tournaments once a month. He has been diagnosed with a stress fracture in one of his vertebrae. Which of the following instructions is most important to emphasize to the boy and his parents? a) "Ice will help reduce the inflammation." b) "NSAIDs can help with pain control and inflammation." c) "You will need to see a physical therapist for stretching and strengthening exercises." d) "You and your coaches need to understand that you cannot play soccer for at least six weeks."

"You and your coaches need to understand that you cannot play soccer for at least six weeks." Correct Explanation: A child with an overuse injury needs to avoid the causative activity for six to eight weeks. The other suggestions are also important, but the nurse must emphasize to the boy and his parents that they must tell the coaches "no soccer for six weeks." In some situations, it is helpful to supply a written directive from the nurse or physician to help the parent avoid undue pressure from coaches.

The nurse is caring for a group of children on the pediatric unit. The nurse should collect further data and explore the possibility of child abuse in which of the following situations? a) A 7-year-old with a spiral fracture of the humerus, which the caregiver reports as having been caused when the child was hit by a bat swung by a Little League teammate. b) A 9-year-old with a compound fracture of the tibia, which the caregiver reports as having been caused when the child attempted a flip on a skateboard. c) A 10-year-old with a simple fracture of the femur, which the caregiver reports as having been caused when the child fell down a set of stairs. d) A 6-year-old with a greenstick fracture of the wrist, which the caregiver reports as having been caused when the child fell while ice-skating.

A 7-year-old with a spiral fracture of the humerus, which the caregiver reports as having been caused when the child was hit by a bat swung by a Little League teammate. Correct Explanation: Spiral fractures, which twist around the bone, are fre quently associated with child abuse and are caused by a wrenching force. When a broken bone penetrates the skin, the fracture is called compound, or open. A simple, or closed, fracture is a single break in the bone without penetration of the skin. In a greenstick fracture, the bone bends and often just partially breaks.

The nurse is caring for a 6-year-old boy with Russell traction applied to his left leg. Which intervention would be most appropriate to prevent complications? a) Adjust the weights as needed. b) Clean and massage his entire leg daily. c) Provide pin care as needed. d) Assess the popliteal region carefully for skin breakdown.

Assess the popliteal region carefully for skin breakdown. Correct Explanation: The nurse would assess the popliteal region carefully for skin breakdown from the sling. The nurse would adjust the weights only per physician orders. Cleaning and massaging the skin is unrelated to care of the child with Russell traction. Russell traction is a form of skin traction, so there is no pin care.

The nurse is observing a 3-year-old boy who is sitting and playing in the waiting area of his pediatrician's office. The nurse calls the boy and his mother back for the boy's appointment. The boy rolls onto his stomach and pushes himself to his knees. Then he presses his hands against his ankles, knees, and thighs, walking up the front of his body, to stand. Which condition should the nurse suspect in this client? a) Duchenne muscular dystrophy b) Congenital myotonic dystrophy c) Juvenile arthritis d) Facioscapulohumeral muscular dystrophy

Duchenne muscular dystrophy Correct Explanation: By age 3, children with Duchenne muscular dystrophy can rise from the floor only by rolling onto their stomachs and then pushing themselves to their knees. To stand, they press their hands against their ankles, knees, and thighs (they "walk up their front"); this is a Gower sign. Symptoms of facioscapulohumeral muscular dystrophy begin after the child is 10 years old, and the primary symptom is facial weakness. The child becomes unable to wrinkle the forehead and cannot whistle. Congenital myotonic dystrophy begins in utero and typically leads to death before age 1 year because of inability to sustain respiratory function. The symptoms of juvenile arthritis are primarily stiff and painful joints.

Fracture of the femur typically occurs when a small child is lifted by one hand, as happens when a parent pulls on one arm to lift the child over a curb or up a step. a) False b) True

False Correct Explanation: If a small child is lifted by one hand, as happens when a parent pulls on one arm to lift the child over a curb or up a step, the head of the radius may escape the ligament surrounding it and become dislocated (nursemaid's elbow). Fracture of the femur is rare and is typically caused by an automobile accident, a fall from a considerable height, or child maltreatment.

The nurse is caring for a child who has just had a plaster cast applied to the arm. The nurse is correct in doing which of the following with this child? a) Using only a draw sheet to move the casted arm. b) Handling the cast with open palms when moving the arm. c) Encouraging the child to move the arm slowly up and down to help the cast dry. d) Keeping a clove-hitch restraint gently tied on the hand to stabilize the arm.

Handling the cast with open palms when moving the arm. Correct Explanation: A wet plaster cast should be handled only with open palms because fingertips can cause indentations and result in pressure points. There is no reason the arm should be restrained or the arm moved to aid in the drying process.

A 5-year-old child is in traction and at risk for impaired skin integrity due to pressure. Which intervention is most effective? a) Keep the child's skin clean and dry b) Gently massage the child's back to stimulate circulation c) Inspect the child's skin for rashes, redness, irritation, or pressure sores d) Apply lotion to dry skin

Inspect the child's skin for rashes, redness, irritation, or pressure sores It is important to be vigilant in inspecting the child's skin for rashes, redness, and irritation to uncover areas where pressure sores are likely to develop. Applying lotion is part of the routine skin care regimen. Applying lotion, gentle massage, and keeping skin dry and clean are part of the routine skin care regimen.

A 2-year-old is diagnosed with osteomyelitis. Which of the following would you anticipate as a primary nursing intervention to include in the child's plan of care? a) Maintaining intravenous antibiotic therapy b) Assisting the child with crutch walking c) Keeping the child quiet while in skeletal traction d) Restricting fluid to encourage red cell production

Maintaining intravenous antibiotic therapy Correct Explanation: Osteomyelitis is a serious infection. It is treated vigorously with intravenous antibiotics. It would not require traction. The stem does not indicate the location of the infection, so the child may not need crutches. Fluid restriction does not help red blood cell production.

A nurse is reviewing the medical record of a child who has sustained a fracture. Documentation reveals a bowing deformity. The nurse interprets this fracture as which of the following? a) Significant bending without actual breaking b) Incomplete fracture c) Bone buckling due to compression d) Bone that breaks into two pieces

Significant bending without actual breaking Correct Explanation: A plastic or bowing deformity is one in which there is significant bending of the bone without breaking. A buckle fracture is one in which the bone buckles rather than breaks. This is usually due to a compression injury. An incomplete fracture of the bone is a greenstick fracture. A complete fracture is one in which the bone breaks into two pieces

A 14-year-old girl with a fractured leg is receiving instructions from the nurse on how to use crutches. Which of the following interventions should the nurse implement to help prevent nerve palsy in the client? a) Be certain the child is walking with the crutches about 6 inches to the side of the foot b) Caution parents to clear articles such as throw rugs out of paths at home c) Teach the client not to rest with the crutch pad pressing on the axilla d) Assess the tips of the crutches to be certain the rubber tip is intact

Teach the client not to rest with the crutch pad pressing on the axilla Correct Explanation: Pressure of a crutch against the axilla could lead to compression and damage of the brachial nerve plexus crossing the axilla, resulting in permanent nerve palsy. Teach children not to rest with the crutch pad pressing on the axilla but always to support their weight at the hand grip. Always assess the tips of crutches to be certain the rubber tip is intact and not worn through as the tip prevents the crutch from slipping. Be certain the child is walking with the crutches placed about 6 inches to the side of the foot. This distance furnishes a wide, balanced base for support. Caution parents to clear articles such as throw rugs, small footstools or toys out of paths at home, to avoid tripping the child.

The nurse is caring for a child diagnosed with Legg-Calvé-Perthes disease. Of the following nursing interventions, which would be most important for the nurse to include in working with this child and the child's caregivers? a) The nurse should support the caregivers in restricting activity during the treatment. b) The nurse should help the caregivers to understand and the child to effectively use the corrective devices. c) The nurse should be a contact person when the child is hospitalized. d) The nurse should provide information when the child or caregiver requests it.

The nurse should help the caregivers to understand and the child to effectively use the corrective devices. Correct Explanation: Nursing care focuses on helping the child and caregivers to manage the corrective device and on the importance of compliance to promote healing and to avoid long-term disability.

In caring for a child in traction, of the following interventions, which is the highest priority for the nurse? a) The nurse should record accurate intake and output. b) The nurse should provide age-appropriate activities for the child. c) The nurse should monitor for decreased circulation every four hours. d) The nurse should clean the pin sites at least once every eight hours.

The nurse should monitor for decreased circulation every four hours. Correct Explanation: Any child in traction must be carefully monitored to detect any signs of decreased circulation or neurovascular complications. Cleaning pin sites is appropriate for a child in skeletal traction. Providing age-appropriate activities and monitoring intake and output are important interventions for any ill child but would not be the highest priority interventions for the child in traction.

A nurse is conducting a physical examination on an 11-year-old boy with Legg-Calvé-Perthes disease. Which assessment finding would be expected? a) Loss of strength in ankle dorsiflexion b) Trendelenburg gait c) Lordosis d) Kyphosis

Trendelenburg gait Correct Explanation: The nurse would expect to note a Trendelenburg gait due to pain. Lordosis is an excessive curvature of the spine and is not associated with Legg-Calvé-Perthes disease. Kyphosis is an excessive curvature of the spine and is not associated with Legg-Calvé-Perthes disease. Loss of strength in ankle dorsiflexion is associated with some neuromuscular disorders but not this condition.

In discussing the treatment for children with scoliosis, a group of pediatric nurses makes the following statements. Which statement is most accurate related to the treatment of scoliosis? a) "Children treated for scoliosis by using braces have to wear the brace almost all the time." b) "Children with severe scoliosis are treated using electrical stimulation." c) "The only successful treatment for scoliosis is surgery within two weeks of the diagnosis." d) "The treatment for children with scoliosis usually lasts three to four months."

a)"Children treated for scoliosis by using braces have to wear the brace almost all the time." Explanation: The Boston brace and the TLSO brace are made of plastic and are customized to fit the child for treatment of scoliosis. The brace should be worn constantly, except during bathing or swimming, to achieve the greatest benefit.

A nurse is setting up an electric heating pad for a 6-year-old boy requiring heat therapy. Which of the following would be the most appropriate way to engage the child during use of the heating pad? a) "Here are some crayons and paper." b) "Look at how the dial glows." c) "Do not unplug the heating pad." d) "Let's see what is on TV."

a)"Here are some crayons and paper." Explanation: It is best to distract the child with an age-appropriate activity. Do not make a show of the set-up process by pointing out the glowing dial or how the device turns on and off. A child might be tempted to turn the heating pad on and off or unplug it, even after being instructed not to. Television is a passive activity and often overused as a distraction for children.

The nurse is reinforcing teaching with the caregivers of a child who has been placed in an external fixation device for the treatment of an orthopedic condition. Which of the following statements made by the caregivers indicate an understanding of the external fixation device? a) "It will be hard, but we know our child will be in this device for a long time." b) "We will have to get some of the elastic bandages to place around the pins and pin sites." c) "If we see any drainage around the pins when we are cleaning them, we won't be concerned." d) "He is very sensitive about the way the device looks, I am glad that his clothes will fully cover it so his friends won't tease him."

a)"It will be hard, but we know our child will be in this device for a long time." Explanation: External fixation devices are sometimes left in place for as long as 1 year. The pin sites are left open to the air and should be inspected and cleansed every 8 hours. The child and caregiver should be able to recognize the signs of infection at the pin sites. The appearance of the pins puncturing the skin and the unusual appearance of the device can be upsetting to the child.

The nurse is teaching a group of peers regarding different types of fractures. Which of the following best describes an open fracture? a) A fracture in which the broken bone penetrates the skin. b) A fracture in which there is a single break in the bone without penetration of the skin. c) A fracture in which the fragments of the bone are separated. d) A fracture in which the fragments of the bone remain partially joined.

a)A fracture in which the broken bone penetrates the skin. Explanation: The open fracture, also called a compound fracture, penetrates the skin. When the fragments of the bone are separated, the fracture is said to be complete.

An infant with a femur fracture is placed in Bryant traction. Which of the following would the nurse include in the infant's plan of care? a) Keeping the buttocks slightly elevated b) Removing the traction boot every 8 hours c) Provide range of motion to the unaffected extremity d) Wrapping the bandages from the ankle to the knee

a)Keeping the buttocks slightly elevated Explanation: With Bryant traction, the buttocks should be slightly elevated and clear of the bed. The bandages are wrapped from the ankles to midthigh in Bryant traction. The legs are wrapped from the ankle to knee. A traction boot is not used with Bryant traction. This action would be appropriate for Buck traction. With Bryant traction, both legs are extended vertically, so range of motion would not be appropriate.

The nurse will teach parents of children with myelomeningocele to maintain an environment free of what element? a) Latex b) Cat dander c) Peanuts d) Alcohol gel

a)Latex A latex-free environment is important because research shows that up to 73% of children with repeated surgeries for spina bifida are sensitive to latex. Those with known sensitivity must be managed in a latex-free environment in the health care setting and in the home, in the school, and beyond. Children at risk for latex sensitivity should wear medical alert identification. The other options may present risks to individual children but are not a threat to those with spina bifida as a group.

A girl with scoliosis is prescribed a body brace. The purpose of the brace is to a) improve spinal alignment. b) prevent torticollis. c) prevent herniation of a spinal disk. d) correct spinal curvature.

a)improve spinal alignment. Explanation: Body bracing helps to hold the spine in alignment and prevent further curvature. The brace will not correct the problem. Herniation and torticollis are not associated with scoliosis.

The emergency department nurse is caring for a 3-year-old girl with an arm injury. The mother is very upset because she believes she broke her daughter's arm. "I was lifting her by her hands and felt a pop in her wrist. She instantly started screaming." The child is now guarding and refusing to move her arm. Which response by the nurse would be most appropriate? a) "You probably dislocated her radial head when you lifted her." b) "Her arm isn't broken. This injury is common and easily fixed with no complications." c) "The popping noise was the ligament surrounding the radial head becoming entrapped." d) "This is most likely nursemaid's elbow; you will have to be more careful in the future."

b)"Her arm isn't broken. This injury is common and easily fixed with no complications." Explanation: The nurse should quickly reassure the mother that this is a common occurrence, seen every day in the emergency department, and is easily fixed and resolves with no complications. Although a popping noise indicates entrapment of the ligament, this response does not address the mother's concerns. Although the radial head most likely dislocated, this response does not address the mother's concern. Although this condition is called nursemaid's elbow, telling the mother she has to be more careful only serves to put blame on the mother and does not address her concerns.

The nurse is caring for an 8-year-old girl in traction. She has been in an acute care setting for two weeks and will require an additional 10 days in the hospital. She is showing signs of regression with thumb sucking and pleas for her tattered baby blanket. Which of the following would be the most helpful intervention? a) "Would you like a coloring book?" b) "Let's ask your mom to bring your friends for a visit." c) "You are too big to suck your thumb." d) "Do you want a book to read?"

b)"Let's ask your mom to bring your friends for a visit." Explanation: After two weeks in traction, a child can become easily bored and regress in social and personal skills. A visit from friends arranged by the girl's mother or supervised by the child-life specialist would help her adapt to her immobilized state. Telling the girl she is too big to suck her thumb is unhelpful. Suggesting a book or coloring book would be unhelpful at this point, as she has likely grown tired of books and coloring after two weeks.

A nurse is treating an 8-year-old boy for supracondylar fracture of the humerus. The boy will be receiving a cast. Which intervention should the nurse implement to prevent Volkmann's ischemic contracture in this client? a) Use of traction to the fingers as a part of the cast b) Apply a cast incompletely for 24 hours, with the elbow being simply splinted c) Cutting a window into the portion of the cast covering the palm d) Teach the client not to rest with the crutch pad pressing on the axilla

b)Apply a cast incompletely for 24 hours, with the elbow being simply splinted Explanation: When an arm is flexed and put into a cast, the radial artery and nerve can be compressed at the elbow, causing nerve injury or severe impairment of circulation. If symptoms of compression are present but not detected within 6 hours, Volkmann's contracture and possible permanent damage to the arm will result. In some instances, a cast is applied incompletely for 24 hours, the elbow portion being simply splinted and wrapped with elastic bandages. After 24 hours, when edema has subsided and the chance of compression is less, the rest of the arm is then casted. Traction to the fingers is used for fractures in the forearm in which the fracture is complete and overriding is excessive. Cutting a window into the portion of the cast covering the palm would not prevent Volkmann's ischemic contracture. Teaching the client not to rest with the crutch pad pressing on the axilla is a measure to prevent nerve palsy in someone using crutches, not to prevent Volkmann's contracture.

The nurse is observing a 3-year-old boy who is sitting and playing in the waiting area of his pediatrician's office. The nurse calls the boy and his mother back for the boy's appointment. The boy rolls onto his stomach and pushes himself to his knees. Then he presses his hands against his ankles, knees, and thighs, walking up the front of his body, to stand. Which condition should the nurse suspect in this client? a) Congenital myotonic dystrophy b) Duchenne muscular dystrophy c) Facioscapulohumeral muscular dystrophy d) Juvenile arthritis

b)Duchenne muscular dystrophy Explanation: By age 3, children with Duchenne muscular dystrophy can rise from the floor only by rolling onto their stomachs and then pushing themselves to their knees. To stand, they press their hands against their ankles, knees, and thighs (they "walk up their front"); this is a Gower sign. Symptoms of facioscapulohumeral muscular dystrophy begin after the child is 10 years old, and the primary symptom is facial weakness. The child becomes unable to wrinkle the forehead and cannot whistle. Congenital myotonic dystrophy begins in utero and typically leads to death before age 1 year because of inability to sustain respiratory function. The symptoms of juvenile arthritis are primarily stiff and painful joints.

A nursing student tells the staff nurse on the pediatric orthopedic unit that she has heard of a musculoskeletal disorder in which there is an infection of the bone. Which of the following disorders does this statement describe? a) Osteosarcoma b) Osteomyelitis c) Muscular dystrophy d) Juvenile rheumatoid arthritis

b)Osteomyelitis Explanation: Osteomyelitis is an infection of the bone usually caused by Staphylococcus aureus. Acute osteomyelitis is twice as common in boys and results from a primary infection.

The nurse is caring for a group of children on the pediatric unit. The nurse should collect further data and explore the possibility of child abuse in which of the following situations? a) A 6-year-old with a greenstick fracture of the wrist, which the caregiver reports as having been caused when the child fell while ice-skating. b) A 10-year-old with a simple fracture of the femur, which the caregiver reports as having been caused when the child fell down a set of stairs. c) A 7-year-old with a spiral fracture of the humerus, which the caregiver reports as having been caused when the child was hit by a bat swung by a Little League teammate. d) A 9-year-old with a compound fracture of the tibia, which the caregiver reports as having been caused when the child attempted a flip on a skateboard.

c)A 7-year-old with a spiral fracture of the humerus, which the caregiver reports as having been caused when the child was hit by a bat swung by a Little League teammate. Explanation: Spiral fractures, which twist around the bone, are fre quently associated with child abuse and are caused by a wrenching force. When a broken bone penetrates the skin, the fracture is called compound, or open. A simple, or closed, fracture is a single break in the bone without penetration of the skin. In a greenstick fracture, the bone bends and often just partially breaks.

Why will it be necessary for the nurse to be very supportive of parents' attempts to feed the infant with recently repaired myelomeningocele? a) Pain will interfere with the feeding process. b) Nausea and vomiting often follow repair of the cystic mass. c) Assuming the usual feeding position will be difficult. d) The infant will have a poor sucking reflex.

c)Assuming the usual feeding position will be difficult. Because the repaired area will need to be protected, having to use an alternate feeding position is likely. The infant may need to be fed prone with the head turned to the side and may not be able to be held. Being able to provide food for the infant is central to parenting the child. Difficulty nurturing a child can be very stressful. Little pain will be experienced and should easily be controlled owing to loss of sensation in the area. The sucking reflex should not be affected by the myelomeningocele or its repair. Nausea and vomiting are unlikely after recovery from the anesthetic.

The nurse is discussing types of treatment used when working with children who have orthopedic disorders. Which of the following forms of treatment covers the lower part of the body, usually from the waist down, and either one or both legs while leaving the feet open? a) Internal fixation device b) Stockinette c) Spica cast d) External fixation device

c)Spica cast Explanation: The hip spica cast covers the lower part of the body, usually from the waist down, and either one or both legs while leaving the feet open. The cast maintains the legs in a frog-like position. Usually, there is a bar placed between the legs to help support the cast.

A 14-year-old girl is diagnosed as having scoliosis. When doing scoliosis screening with her, an important observation would be to note a) the angle of the iliac crest when she bends forward. b) the angle of her lower chest when she sits down. c) her posterior spine when she bends forward. d) the posterior spine when she bends sideways.

c)her posterior spine when she bends forward. Explanation: A lateral curvature of the spine (scoliosis) is best revealed when the child bends forward. Bending to the side would not provide an accurate assessment of the spine nor would assessing the iliac crest or the chest.

While an adolescent wears a body brace for scoliosis, you would teach her a) to wear the brace a maximum of 20 hours each day. b) that secondary sex changes will stop until the brace is removed. c) to continue with age-appropriate activities. d) to stand absolutely still whenever she is out of the brace.

c)to continue with age-appropriate activities. Explanation: Wearing a body brace should not interfere with normal activities, which are necessary to maintain adolescent self-esteem. Sex changes continue with or without bracing; the provider will determine the length of time for wearing the brace each day.

A 3-year-old demonstrates lateral bowing of the tibia. Which of the following signs would indicate that the boy's condition is Blount disease rather than just the more typical developmental genu varum? a) The condition is bilateral b) The medial surfaces of the knees are more than 2 in apart c) The malleoli are touching d) A sharp, beaklike appearance to the medial aspect of the proximal tibia on x-ray

d)A sharp, beaklike appearance to the medial aspect of the proximal tibia on x-ray Explanation: Blount disease is retardation of growth of the epiphyseal line on the medial side of the proximal tibia (inside of the knee) that results in bowed legs. Unlike the normal developmental aspect of genu varum, Blount disease is usually unilateral and is a serious disturbance in bone growth that requires treatment. In those with Blount disease, the medial aspect of the proximal tibia will show a sharp, beaklike appearance. The other answers all describe genu varum, not Blount disease.

In understanding the development of the musculoskeletal system, the nurse recognizes that which of the following is implanted in a gel-like substance during fetal life? a) Ligaments b) Tendons c) Joints d) Cartilage

d)Cartilage Explanation: During fetal life, tissue called cartilage, which is a type of connective tissue consisting of cells implanted in a gel-like substance, gradually calcifies and becomes bone.

A nurse who is discussing Duchenne muscular dystrophy characterizes it correctly using which descriptors? a) Duchenne muscular dystrophy is a progressive disease of muscles and nerves that affects males and females equally. b) Duchenne muscular dystrophy is diagnosed in boys who develop gait changes during the late school-age years. c) Duchenne muscular dystrophy is a nonprogressive disorder that severely affects muscle function through spinal cord atrophy. d) Duchenne muscular dystrophy causes progressive muscular weakness that ends in death.

d)Duchenne muscular dystrophy causes progressive muscular weakness that ends in death. Duchenne muscular dystrophy is the most common of several muscular dystrophies and is a progressive, fatal disorder. It involves mainly skeletal muscles, but other muscles are affected over time. Onset occurs in early childhood. The disorder is X-linked recessive. An enzyme is lacking that is necessary for the maintenance of muscle cells. No structural abnormalities of the spinal cord or peripheral nerves are noted.

The nurse is discussing fractures seen in children with a group of caregivers. One of the caregivers states, "I have heard that if a bone breaks it can cause permanent damage and stop the growth of the bone." This statement is accurate if the break occurs in which of the following? a) Xiphoid process b) Joint c) Humerus d) Epiphyseal plate

d)Epiphyseal plate Explanation: Fractures in the area of the epiphyseal plate (growth plate) can cause permanent damage and severely impair growth

A 2-year-old is diagnosed with osteomyelitis. Which of the following would you anticipate as a primary nursing intervention to include in the child's plan of care? a) Assisting the child with crutch walking b) Restricting fluid to encourage red cell production c) Keeping the child quiet while in skeletal traction d) Maintaining intravenous antibiotic therapy

d)Maintaining intravenous antibiotic therapy Explanation: Osteomyelitis is a serious infection. It is treated vigorously with intravenous antibiotics. It would not require traction. The stem does not indicate the location of the infection, so the child may not need crutches. Fluid restriction does not help red blood cell production.

A nursing instructor is preparing a class presentation about tibia vara. Which of the following would the instructor include as a risk factor? a) Late walking b) Lack of sunlight exposure c) Hormonal alterations during puberty d) Obesity

d)Obesity Explanation: Obesity is a risk factor for the development of tibia vara. Tibia vara occurs most frequently in children who are early walkers. Limited or lack of exposure to sunlight may lead to rickets. Hormonal alterations during puberty may play a role in the development of slipped capital femoral epiphysis.

Which nursing diagnosis is most relevant in the first 12 hours of life for a neonate born with a myelomeningocele? a) Impaired physical mobility b) Constipation c) Delayed growth and development d) Risk for infection

d)Risk for infection Explanation: All of these diagnoses are important for a child with a myelomeningocele. However, during the first 12 hours of life, the most life-threatening event would be an infection. The other diagnoses will be addressed as the child develops.

A nurse is conducting a physical examination on an 11-year-old boy with Legg-Calvé-Perthes disease. Which assessment finding would be expected? a) Lordosis b) Loss of strength in ankle dorsiflexion c) Kyphosis d) Trendelenburg gait

d)Trendelenburg gait Explanation: The nurse would expect to note a Trendelenburg gait due to pain. Lordosis is an excessive curvature of the spine and is not associated with Legg-Calvé-Perthes disease. Kyphosis is an excessive curvature of the spine and is not associated with Legg-Calvé-Perthes disease. Loss of strength in ankle dorsiflexion is associated with some neuromuscular disorders but not this condition.

A 14-year-old girl is diagnosed as having scoliosis. When doing scoliosis screening with her, an important observation would be to note a) her posterior spine when she bends forward. b) the angle of the iliac crest when she bends forward. c) the posterior spine when she bends sideways. d) the angle of her lower chest when she sits down.

her posterior spine when she bends forward. Correct Explanation: A lateral curvature of the spine (scoliosis) is best revealed when the child bends forward. Bending to the side would not provide an accurate assessment of the spine nor would assessing the iliac crest or the chest.

The nurse performing a focused health history on a newborn asks the parents if there are any hereditary disorders affecting musculoskeletal function in the family history. These disorders include (select all answers that apply): a) Clubfoot b) Scoliosis c) Hip dysplasia d) Limb deformities e) Brachial plexus injury

• Scoliosis • Clubfoot • Hip dysplasia Explanation: The nurse should explore the family history for any hereditary disorders. The presence of scoliosis, clubfoot, hip or skeletal dysplasia, or neuromuscular disorders in family members may help in diagnosing genetically linked orthopedic disorders.

In understanding the function of the musculoskeletal system, the nurse recognizes that which of the following allows for movement of the body parts? a) Joints b) Tendons c) Ligaments d) Cartilage

Joints Correct Explanation: Bones are attached to each other by connecting links called joints, which allow for movement of the body parts. Skeletal muscles attach to the bones, with a moveable joint between them. Tendons and ligaments hold the muscles and bones together. Cartilage is a type of connective tissue consisting of cells implanted in a gel-like substance, which gradually calcifies and becomes bone.

The type of traction in which a pin, wire, tongs, or other device is surgically inserted through a bone is which of the following? a) Russell traction b) Skeletal traction c) Buck extension traction d) Skin traction

Skeletal traction Correct Explanation: Skeletal traction exerts pull directly on skeletal structures by means of a pin, wire, tongs, or other device surgically inserted through a bone. Skin traction applies pull on tape, rubber, or a plastic material attached to the skin, which indirectly exerts pull on the musculoskeletal system. Examples of skin traction are Bryant's traction, Buck extension traction, and Russell traction.

The nurse is working with an 8-year-old girl who recently developed juvenile arthritis. The mother of the girl tells the nurse that she understands that exercise is important to help preserve muscle and joint function and asks the nurse for recommendations on types of exercise that would be appropriate. Which of the following should the nurse recommend? a) Jumping jacks b) Swimming c) Hiking d) Soccer

Swimming Correct Explanation: Swimming and tricycle or bicycle riding are excellent exercises because they provide smooth joint action. In contrast, to reduce joint destruction, activities that place excessive strain on joints, such as running, jumping, prolonged walking, and kicking, should be avoided.

The young boy has fractured his left leg and has had a cast applied. The nurse educates the boy and his parents prior to discharge from the hospital. The parents should call the physician when which incidents occur? - New drainage is seeping out from under the cast. - The outside of the boy's cast got wet and had to be dried using a hair dryer. - The boy has had a fever of greater than 102° F (38.9°C) for the last 36 hours. - The boy experiences mild pain when wiggling his toes. - The boy's toes are light blue and very swollen.

The boy has had a fever of greater than 102° F (38.9°C) for the last 36 hours. • New drainage is seeping out from under the cast. • The boy's toes are light blue and very swollen. The parents should call the physician when the following things occur: The child has a temperature greater than 101.5F° (38.7° C) for more than 24 hours, there is drainage from the casted site, the site distal to the casted extremity is cyanotic, or severe edema is present.

The nurse is presenting an in-service to a group of peers on the topic of traction. The nurse asks the group to give examples of types of skin traction. The following types were named by the nurses. Which of the following is an example of a type of skeletal traction? a) Russell traction b) Balanced suspension traction c) Bryant's traction d) Buck extension traction

b)Balanced suspension traction Explanation: Skeletal traction exerts pull directly on skeletal structures by means of a pin, wire, tongs, or other device surgically inserted through a bone. Examples of skeletal traction are 90-degree traction and balanced suspension traction. Dunlop's traction, sometimes used for fractures of the humerus or the elbow, can be either skin or skeletal traction. It is skeletal traction if a pin is inserted into the bone to immobilize the extremity. Skin traction applies pull on tape, rubber, or a plastic material attached to the skin, which indirectly exerts pull on the musculoskeletal system. Examples of skin traction are Bryant's traction, Buck extension traction, and Russell traction.

In caring for a child in traction, of the following interventions, which is the highest priority for the nurse? a) The nurse should clean the pin sites at least once every eight hours. b) The nurse should monitor for decreased circulation every four hours. c) The nurse should record accurate intake and output. d) The nurse should provide age-appropriate activities for the child.

b)The nurse should monitor for decreased circulation every four hours. Explanation: Any child in traction must be carefully monitored to detect any signs of decreased circulation or neurovascular complications. Cleaning pin sites is appropriate for a child in skeletal traction. Providing age-appropriate activities and monitoring intake and output are important interventions for any ill child but would not be the highest priority interventions for the child in traction.

The nurse is caring for a 14-year-old boy in Buck traction for a slipped capital femoral epiphysis (SCFE). Which of the following would the nurse include when completing a neurovascular assessment of the affected leg? Select all that apply. a) Sensation b) Capillary refill c) Color d) Vital signs e) Pulse

c)Color a)Sensation e)Pulse b)Capillary refill Explanation: A neurovascular assessment includes assessing for color, movement, sensation, edema, and quality of pulses. Vital signs are not a component of a neurovascular assessment.

A nurse is performing a physical examination of a child with a suspected fracture. Which assessment technique would the nurse be least likely to use? a) Observation b) Palpation c) Inspection d) Auscultation

d)Auscultation Explanation: The physical examination specific to fractures includes inspection, observation, and palpation. Auscultation is not used.

A nurse is reviewing the medical record of a child who has sustained a fracture. Documentation reveals a bowing deformity. The nurse interprets this fracture as which of the following? a) Bone that breaks into two pieces b) Incomplete fracture c) Bone buckling due to compression d) Significant bending without actual breaking

d)Significant bending without actual breaking Explanation: A plastic or bowing deformity is one in which there is significant bending of the bone without breaking. A buckle fracture is one in which the bone buckles rather than breaks. This is usually due to a compression injury. An incomplete fracture of the bone is a greenstick fracture. A complete fracture is one in which the bone breaks into two pieces.

The nurse is presenting an in-service to a group of peers on the topic of traction. The nurse asks the group to give examples of types of skin traction. The following types were named by the nurses. Which of the following is an example of a type of skeletal traction? a) Balanced suspension traction b) Bryant's traction c) Russell traction d) Buck extension traction

Balanced suspension traction Correct Explanation: Skeletal traction exerts pull directly on skeletal structures by means of a pin, wire, tongs, or other device surgically inserted through a bone. Examples of skeletal traction are 90-degree traction and balanced suspension traction. Dunlop's traction, sometimes used for fractures of the humerus or the elbow, can be either skin or skeletal traction. It is skeletal traction if a pin is inserted into the bone to immobilize the extremity. Skin traction applies pull on tape, rubber, or a plastic material attached to the skin, which indirectly exerts pull on the musculoskeletal system. Examples of skin traction are Bryant's traction, Buck extension traction, and Russell traction.

When performing physical assessments of children with musculoskeletal disorders, the nurse distinguishes normal variations in children's muscles versus adult muscles. These variations include: a) The infant's muscles account for 45% of total body weight as opposed to 25% of adult body weight. b) During adolescence, muscle growth is influenced by increased production of androgenic hormones. c) The young child has rigid soft tissue, so dislocations and sprains are common occurrences. d) Rapid bone and muscle growth in adolescents increase their agility, thereby decreasing the incidence of injuries.

During adolescence, muscle growth is influenced by increased production of androgenic hormones. Correct Explanation: During adolescence, muscle growth is influenced by hormonal changes, primarily the increased production of androgenic hormones. The infant's muscles account for only 25% of total body weight, whereas they account for 40% to 45% in an adult. The young child has resilient soft tissue, so dislocations and sprains are unusual occurrences. Rapid bone and muscle growth may contribute to the appearance of "clumsy" and awkward motions of the adolescent who is trying to adjust to new body dimensions.

A group of students are reviewing information about the skeletal development in children. The students demonstrate understanding of the information when they identify that ossification is complete by what age? a) Adolescence b) School age c) Toddlerhood d) Preschool age

a)Adolescence Explanation: Ossification and conversion of cartilage to bone continue throughout childhood and are complete at adolescence.

You assist with the application of a full-body plaster cast to a child. The child immediately becomes diaphoretic and complains of being hot. Which nursing intervention would be indicated? a) Advise the child that this is to be expected. b) Observe the child for infection. c) Suggest removal of the cast to the orthopedist. d) Moisten the cast with cool water.

a)Advise the child that this is to be expected. Explanation: Plaster becomes hot as it sets. This effect is reduced with newer plastic casts. This is a normal expectation about which to educate the child before the application of the cast. If discomfort continues, notify the provider. Infection would not present in this way with a cast application. Never moisten a case.

What will be the nurse's next action after noting dimpling and a tuft of hair located in the lumbosacral area of the preschool child during examination? a) Record and refer the finding for follow-up to the pediatrician b) Move on to other assessments without calling attention to the difference c) Snip the tuft of hair off close to the skin for hygienic reasons d) Inspect for precocious hair growth in the genital and underarm areas

a)Record and refer the finding for follow-up to the pediatrician Dimpling and hair growth may signal spina bifida occulta, which usually is benign. However, some complications can be associated, and further investigation is warranted to prevent possible damage to the spinal cord. Magnetic resonance imaging (MRI) is often the diagnostic tool used. No hygienic concerns need prevail. These findings do not suggest development of precocious puberty or any other hormonal problem. The dimpling and hair tuft must be clearly explained to the parents.

The nurse is caring for a child diagnosed with Legg-Calvé-Perthes disease. Of the following nursing interventions, which would be most important for the nurse to include in working with this child and the child's caregivers? a) The nurse should help the caregivers to understand and the child to effectively use the corrective devices. b) The nurse should support the caregivers in restricting activity during the treatment. c) The nurse should provide information when the child or caregiver requests it. d) The nurse should be a contact person when the child is hospitalized.

a)The nurse should help the caregivers to understand and the child to effectively use the corrective devices. Explanation: Nursing care focuses on helping the child and caregivers to manage the corrective device and on the importance of compliance to promote healing and to avoid long-term disability.

The nurse is talking with the caregiver of a 13-year-old diagnosed with scoliosis. The child has come to the clinic to be fitted with a brace to begin her treatment. The child appears upset and angry and states, "I hate this brace; I hate it already." In an effort to support this child, which of the following statements would be the most appropriate for the nurse to make to this child's caregiver? a) "Remind your child that her spine needs to be corrected in order to keep her whole musculoskeletal system healthy for a long, long time." b) "If you can afford it, let your daughter choose an article or two of clothing that she can wear with the brace that will help her feel that she looks good." c) "Children her age often withdraw during stressful times; let her have some time alone to think about the situation and to get used to the brace." d) "Take your daughter to an oncology floor for a few minutes so she can see children who are much sicker than she is."

b)"If you can afford it, let your daughter choose an article or two of clothing that she can wear with the brace that will help her feel that she looks good." Explanation: Help the child select clothing that blends with current styles but is loose enough to hide the brace. Self-image and the need to be like others are very important at this age. Wearing a brace creates a distinct change in body image, especially in the older child or adolescent, at a time when body consciousness is at an all-time high. The need to wear the brace and deal with the limitations it involves may cause anger; the change in body image can cause a grief reaction. Handling these feelings successfully requires understanding support from the nurse, family, and peers. It is important for the child to have an opportunity to talk about his or her feelings.

After teaching the parents of a 6-year-old child about caring for a sprained wrist, which statement by the parents indicates the need for additional teaching? a) "She'll need to limit any activity that involves the wrist." b) "We'll apply a warm moist compress to the wrist for 20 minutes at a time." c) "We'll make sure she keeps her arm above heart level." d) "We can wrap the wrist in an elastic bandage to help reduce the swelling."

b)"We'll apply a warm moist compress to the wrist for 20 minutes at a time." Explanation: Care for a sprain includes rest, ice, compression, and elevation. Cold therapy, not heat, is used for 20 to 30 minutes at a time, then removed for 1 hour and repeated for the first 24 to 48 hours. Compression via an elastic bandage, elevating above heart level, and limiting activity are appropriate measures.

The nurse is working with a group of caregivers of school-age children discussing fractures. The nurse explains that if the fragments of fractured bone are separated, the fracture is said to be which of the following? a) Incomplete b) Complete c) Spiral d) Greenstick

b)Complete Explanation: If the fragments of fractured bone are separated, the fracture is said to be complete. If fragments remain partially joined, the fracture is termed incomplete. Green stick fractures are one kind of incomplete fracture, caused by incomplete ossification, common in children. Spiral fractures twist around the bone.

The nurse is doing patient teaching with a child who has been placed in a brace to treat scoliosis. Which of the following statements made by the child indicates an understanding of the treatment? a) Wearing this brace only during the night won't be so embarrassing." b) "I am so glad I can take this brace off for the school dance." c) "At least when I take a shower I have a few minutes out of this brace." d) "When I start feeling tired, I can just take my brace off for a few minutes."

c)"At least when I take a shower I have a few minutes out of this brace." Explanation: The brace worn to treat scoliosis is worn day and night and should be removed only very briefly, such as for showering. The child needs to be taught that the brace must be worn at all times, during the day as well as the night.

The nurse is caring for a child recently fitted with braces on both legs due to cerebral palsy (CP). Which of the following would the nurse emphasize in the discharge teaching? a) "It is very important to comply with the use of this brace." b) "If the brace is painful, feel free to take it off." c) "Check the skin that is covered by the braces for redness and breakdown." d) "Please try and follow the therapist's on and off schedule."

c)"Check the skin that is covered by the braces for redness and breakdown." Assessing skin integrity should be the priority, as braces can lead to pressure ulcers and infection. Compliance is important, but attention to skin care is the priority teaching. Following the schedule is important for compliance, but skin integrity is the priority. Advising the parents to remove the brace if it is painful is inaccurate; the child may require pain management or further consultation with the physical therapist.

The nurse is teaching the parents of a female child with a myelomeningocele how to perform clean intermittent catheterization. The nurse determines that the teaching was effective when the parents return demonstrate the procedure and state which of the following? a) "Before inserting the catheter, we need to wipe her labia with normal saline from back to front." b) "We need to apply some petroleum jelly to her labia and the catheter before we attempt to insert it." c) "When the urine stops flowing, we should press on the lower belly to ensure the bladder is empty." d) "We need to insert the catheter about 6 inches so that we make sure the catheter is in the bladder."

c)+"When the urine stops flowing, we should press on the lower belly to ensure the bladder is empty." : When the urine stops flowing, the parents should press on the lower abdomen or have the child lean forward to tense the abdominals to ensure that no more urine is in the bladder. For a female, the catheter is inserted about 2 to 3 inches. For a male, the catheter is inserted about 4 to 6 inches. Before the catheter is inserted, the labia is cleaned with a washcloth or disposable wipe from front to back. A generous amount of water-soluble lubricant, not petroleum jelly, is applied to the catheter. There is no need to apply the lubricant to the labia.

The nurse is caring for a child with osteomyelitis. Which of the following is true regarding this diagnosis? Select all that apply. a) As abscess forms, ruptures and spreads infection in the metaphysis of the bone b) Corticosteroids are the treatment of choice c) Transmission-based precautions are followed in most cases d) Caused by Staphylococcus aureus e) Lab findings show a leukocyte count of 1,000 to 2,000 cells

d)Caused by Staphylococcus aureus c)Transmission-based precautions are followed in most cases a)As abscess forms, ruptures and spreads infection in the metaphysis of the bone Explanation: Osteomyelitis is caused by Staphylococcus aureus. The bacteria enter the bloodstream and are carried to the metaphysis, the growing portion of the bone, where an abscess forms, ruptures, and spreads the infection along the bone under the periosteum. Transmission-based precautions may be required if a wound is open and draining.

A nurse is working with a 12-year-old girl with osteomyelitis who is recovering from surgery. Which of the following are nursing interventions that should be implemented in this case? *(Select all that apply.) a) Instituting infection-control precautions related to drainage tubes b) Instruction to the parents regarding the importance of the child maintaining bed rest c) Casting of the affected limb d) Instruction to the parents regarding how to care for an antibiotic IV line at home e) Administration of IV antibiotics at the hospital f) Instruction to the parents regarding proper traction of the limb

e)Administration of IV antibiotics at the hospital b)Instruction to the parents regarding how to care for an antibiotic IV line at home b)Instruction to the parents regarding the importance of the child maintaining bed rest a)Instituting infection-control precautions related to drainage tubes Explanation: Osteomyelitis is infection of the bone. Medical therapy includes limitation of weight bearing on the affected part, bed rest, immobilization, and a short administration of an IV antibiotic such as oxacillin (Bactocill), as indicated by the blood culture. Intravenous therapy is usually initiated in the hospital and then continued at home for as long as 2 weeks. When the child is discharged from the hospital, be certain to review with parents measures to care for the antibiotic intravenous line if this will be continued at home. Keep in mind young children are active, even if they are on bed rest so need age appropriate activities so they maintain rest, not activity. If a child had surgery and drainage tubes are in place, institute infection-control precautions, because the drain evacuates infected material. Neither casting nor traction is required for osteomyelitis.

The nurse is caring for a child after an accident in which the child fractured his arm. A cast has been applied to the child's right arm. Which of the following actions should the nurse implement? Select all that apply. a) Wear sterile gloves when removing or touching the cast. b) Monitor the color of the nail beds in the right hand. c) Check radial pulse in the both arms. d) Document any signs of pain. e) Wear a protective gown when moving the child's arm.

• Monitor the color of the nail beds in the right hand. • Check radial pulse in the both arms. • Document any signs of pain. Correct Explanation: Monitoring for signs of pain, decreased circulation, or change or variation in pulses in the extremity is important for the child in a cast. Pain can indicate serious complications, such as compartment syndrome. Wearing a gown or sterile gloves is unnecessary. Checking posterior pulses would be appropriate when a lower extremity is casted.


Set pelajaran terkait

Compensation Chapter 9 Study Guide

View Set

EMT Chapter 30: Injuries to the Abdomen and Genitalia

View Set

Points, Lines, Planes, Midpoint, Distance

View Set